How to prove that $a=a',b' Rightarrow a=a'=b' $? Announcing the arrival of Valued Associate #679: Cesar Manara Planned maintenance scheduled April 17/18, 2019 at 00:00UTC (8:00pm US/Eastern)Set theory like first-order theory of ordered pairsSome questions regarding set theory.A question regarding the validity of the Union AxiomComprehension and ImpredicativityAxiom of pairing finite structureThe natural numbers as the intersection of all inductive setsA question about collections that agree with logical statements in $ZF$Equivalent statement for theAxiom of PairingAre notations in ZF conservative?How to prove statements in formal set theory? Substitution, the empty set and an example.

Can a non-EU citizen traveling with me come with me through the EU passport line?

What causes the vertical darker bands in my photo?

Seeking colloquialism for “just because”

How can I make names more distinctive without making them longer?

Denied boarding although I have proper visa and documentation. To whom should I make a complaint?

List *all* the tuples!

Why are Kinder Surprise Eggs illegal in the USA?

How does the particle を relate to the verb 行く in the structure「A を + B に行く」?

Check which numbers satisfy the condition [A*B*C = A! + B! + C!]

How to align text above triangle figure

Can a USB port passively 'listen only'?

Why did the rest of the Eastern Bloc not invade Yugoslavia?

Apollo command module space walk?

Dating a Former Employee

What is the role of the transistor and diode in a soft start circuit?

ListPlot join points by nearest neighbor rather than order

List of Python versions

What is Arya's weapon design?

Do I really need recursive chmod to restrict access to a folder?

51k Euros annually for a family of 4 in Berlin: Is it enough?

Storing hydrofluoric acid before the invention of plastics

How come Sam didn't become Lord of Horn Hill?

String `!23` is replaced with `docker` in command line

The logistics of corpse disposal



How to prove that $a=a',b' Rightarrow a=a'=b' $?



Announcing the arrival of Valued Associate #679: Cesar Manara
Planned maintenance scheduled April 17/18, 2019 at 00:00UTC (8:00pm US/Eastern)Set theory like first-order theory of ordered pairsSome questions regarding set theory.A question regarding the validity of the Union AxiomComprehension and ImpredicativityAxiom of pairing finite structureThe natural numbers as the intersection of all inductive setsA question about collections that agree with logical statements in $ZF$Equivalent statement for theAxiom of PairingAre notations in ZF conservative?How to prove statements in formal set theory? Substitution, the empty set and an example.










1












$begingroup$


I'm using Jean-Louis Krivine's book "Theorie des ensembles" as a reference book so the notations might be a little different from english literature, but i guess the question is easy to follow.



Let $mathcal U$ be a universe (a collection of objects that is supposedly not empty, the objects are called sets). We define a binary relation on this universe that we note "$in$".



Axiom of extensionality: $forall x forall y[forall z(zin xLeftrightarrow z in y)Rightarrow x=y]$.



Axiom of pairing: $forall x forall y exists zforall t[tin zLeftrightarrow (t=x lor t=y)]$.



My question is, assuming only these two axioms, how to prove that if we had sets $a,a' and b'$ satisfying $a=a',b'$ then $ a=a'=b' $?










share|cite|improve this question











$endgroup$
















    1












    $begingroup$


    I'm using Jean-Louis Krivine's book "Theorie des ensembles" as a reference book so the notations might be a little different from english literature, but i guess the question is easy to follow.



    Let $mathcal U$ be a universe (a collection of objects that is supposedly not empty, the objects are called sets). We define a binary relation on this universe that we note "$in$".



    Axiom of extensionality: $forall x forall y[forall z(zin xLeftrightarrow z in y)Rightarrow x=y]$.



    Axiom of pairing: $forall x forall y exists zforall t[tin zLeftrightarrow (t=x lor t=y)]$.



    My question is, assuming only these two axioms, how to prove that if we had sets $a,a' and b'$ satisfying $a=a',b'$ then $ a=a'=b' $?










    share|cite|improve this question











    $endgroup$














      1












      1








      1





      $begingroup$


      I'm using Jean-Louis Krivine's book "Theorie des ensembles" as a reference book so the notations might be a little different from english literature, but i guess the question is easy to follow.



      Let $mathcal U$ be a universe (a collection of objects that is supposedly not empty, the objects are called sets). We define a binary relation on this universe that we note "$in$".



      Axiom of extensionality: $forall x forall y[forall z(zin xLeftrightarrow z in y)Rightarrow x=y]$.



      Axiom of pairing: $forall x forall y exists zforall t[tin zLeftrightarrow (t=x lor t=y)]$.



      My question is, assuming only these two axioms, how to prove that if we had sets $a,a' and b'$ satisfying $a=a',b'$ then $ a=a'=b' $?










      share|cite|improve this question











      $endgroup$




      I'm using Jean-Louis Krivine's book "Theorie des ensembles" as a reference book so the notations might be a little different from english literature, but i guess the question is easy to follow.



      Let $mathcal U$ be a universe (a collection of objects that is supposedly not empty, the objects are called sets). We define a binary relation on this universe that we note "$in$".



      Axiom of extensionality: $forall x forall y[forall z(zin xLeftrightarrow z in y)Rightarrow x=y]$.



      Axiom of pairing: $forall x forall y exists zforall t[tin zLeftrightarrow (t=x lor t=y)]$.



      My question is, assuming only these two axioms, how to prove that if we had sets $a,a' and b'$ satisfying $a=a',b'$ then $ a=a'=b' $?







      elementary-set-theory logic






      share|cite|improve this question















      share|cite|improve this question













      share|cite|improve this question




      share|cite|improve this question








      edited Mar 26 at 16:52









      Mauro ALLEGRANZA

      68.1k449117




      68.1k449117










      asked Mar 26 at 16:43









      Matsukazi IciMatsukazi Ici

      397




      397




















          2 Answers
          2






          active

          oldest

          votes


















          4












          $begingroup$

          We have that $a' in a',b′ = a $ and $b' in a',b′ = a $.



          But $a' in a $ means that $a'=a$, and the same with $b'$.



          Thus, from transitivity of equality:




          $a'=a=b'$.







          share|cite|improve this answer











          $endgroup$












          • $begingroup$
            My bad, i thought that $a=a′,b′$ means that $ a' in a,$ which is false, and instead we have $ a' in a$, and $a$ being the only set containig $a$ and just $a$ gives $a'=a$. thank you so much for your answer.
            $endgroup$
            – Matsukazi Ici
            Mar 26 at 20:07



















          1












          $begingroup$

          Hint: If you don't like the direct route you can go by contradiction: Suppose $a neq a'$, for example. Then use pairing to obtain the sets $ a $ and $ a',b' $. Now use the contrapositive of extensionality; what do you get?






          share|cite|improve this answer









          $endgroup$













            Your Answer








            StackExchange.ready(function()
            var channelOptions =
            tags: "".split(" "),
            id: "69"
            ;
            initTagRenderer("".split(" "), "".split(" "), channelOptions);

            StackExchange.using("externalEditor", function()
            // Have to fire editor after snippets, if snippets enabled
            if (StackExchange.settings.snippets.snippetsEnabled)
            StackExchange.using("snippets", function()
            createEditor();
            );

            else
            createEditor();

            );

            function createEditor()
            StackExchange.prepareEditor(
            heartbeatType: 'answer',
            autoActivateHeartbeat: false,
            convertImagesToLinks: true,
            noModals: true,
            showLowRepImageUploadWarning: true,
            reputationToPostImages: 10,
            bindNavPrevention: true,
            postfix: "",
            imageUploader:
            brandingHtml: "Powered by u003ca class="icon-imgur-white" href="https://imgur.com/"u003eu003c/au003e",
            contentPolicyHtml: "User contributions licensed under u003ca href="https://creativecommons.org/licenses/by-sa/3.0/"u003ecc by-sa 3.0 with attribution requiredu003c/au003e u003ca href="https://stackoverflow.com/legal/content-policy"u003e(content policy)u003c/au003e",
            allowUrls: true
            ,
            noCode: true, onDemand: true,
            discardSelector: ".discard-answer"
            ,immediatelyShowMarkdownHelp:true
            );



            );













            draft saved

            draft discarded


















            StackExchange.ready(
            function ()
            StackExchange.openid.initPostLogin('.new-post-login', 'https%3a%2f%2fmath.stackexchange.com%2fquestions%2f3163450%2fhow-to-prove-that-a-a-b-rightarrow-a-a-b%23new-answer', 'question_page');

            );

            Post as a guest















            Required, but never shown

























            2 Answers
            2






            active

            oldest

            votes








            2 Answers
            2






            active

            oldest

            votes









            active

            oldest

            votes






            active

            oldest

            votes









            4












            $begingroup$

            We have that $a' in a',b′ = a $ and $b' in a',b′ = a $.



            But $a' in a $ means that $a'=a$, and the same with $b'$.



            Thus, from transitivity of equality:




            $a'=a=b'$.







            share|cite|improve this answer











            $endgroup$












            • $begingroup$
              My bad, i thought that $a=a′,b′$ means that $ a' in a,$ which is false, and instead we have $ a' in a$, and $a$ being the only set containig $a$ and just $a$ gives $a'=a$. thank you so much for your answer.
              $endgroup$
              – Matsukazi Ici
              Mar 26 at 20:07
















            4












            $begingroup$

            We have that $a' in a',b′ = a $ and $b' in a',b′ = a $.



            But $a' in a $ means that $a'=a$, and the same with $b'$.



            Thus, from transitivity of equality:




            $a'=a=b'$.







            share|cite|improve this answer











            $endgroup$












            • $begingroup$
              My bad, i thought that $a=a′,b′$ means that $ a' in a,$ which is false, and instead we have $ a' in a$, and $a$ being the only set containig $a$ and just $a$ gives $a'=a$. thank you so much for your answer.
              $endgroup$
              – Matsukazi Ici
              Mar 26 at 20:07














            4












            4








            4





            $begingroup$

            We have that $a' in a',b′ = a $ and $b' in a',b′ = a $.



            But $a' in a $ means that $a'=a$, and the same with $b'$.



            Thus, from transitivity of equality:




            $a'=a=b'$.







            share|cite|improve this answer











            $endgroup$



            We have that $a' in a',b′ = a $ and $b' in a',b′ = a $.



            But $a' in a $ means that $a'=a$, and the same with $b'$.



            Thus, from transitivity of equality:




            $a'=a=b'$.








            share|cite|improve this answer














            share|cite|improve this answer



            share|cite|improve this answer








            edited Mar 26 at 19:14

























            answered Mar 26 at 16:51









            Mauro ALLEGRANZAMauro ALLEGRANZA

            68.1k449117




            68.1k449117











            • $begingroup$
              My bad, i thought that $a=a′,b′$ means that $ a' in a,$ which is false, and instead we have $ a' in a$, and $a$ being the only set containig $a$ and just $a$ gives $a'=a$. thank you so much for your answer.
              $endgroup$
              – Matsukazi Ici
              Mar 26 at 20:07

















            • $begingroup$
              My bad, i thought that $a=a′,b′$ means that $ a' in a,$ which is false, and instead we have $ a' in a$, and $a$ being the only set containig $a$ and just $a$ gives $a'=a$. thank you so much for your answer.
              $endgroup$
              – Matsukazi Ici
              Mar 26 at 20:07
















            $begingroup$
            My bad, i thought that $a=a′,b′$ means that $ a' in a,$ which is false, and instead we have $ a' in a$, and $a$ being the only set containig $a$ and just $a$ gives $a'=a$. thank you so much for your answer.
            $endgroup$
            – Matsukazi Ici
            Mar 26 at 20:07





            $begingroup$
            My bad, i thought that $a=a′,b′$ means that $ a' in a,$ which is false, and instead we have $ a' in a$, and $a$ being the only set containig $a$ and just $a$ gives $a'=a$. thank you so much for your answer.
            $endgroup$
            – Matsukazi Ici
            Mar 26 at 20:07












            1












            $begingroup$

            Hint: If you don't like the direct route you can go by contradiction: Suppose $a neq a'$, for example. Then use pairing to obtain the sets $ a $ and $ a',b' $. Now use the contrapositive of extensionality; what do you get?






            share|cite|improve this answer









            $endgroup$

















              1












              $begingroup$

              Hint: If you don't like the direct route you can go by contradiction: Suppose $a neq a'$, for example. Then use pairing to obtain the sets $ a $ and $ a',b' $. Now use the contrapositive of extensionality; what do you get?






              share|cite|improve this answer









              $endgroup$















                1












                1








                1





                $begingroup$

                Hint: If you don't like the direct route you can go by contradiction: Suppose $a neq a'$, for example. Then use pairing to obtain the sets $ a $ and $ a',b' $. Now use the contrapositive of extensionality; what do you get?






                share|cite|improve this answer









                $endgroup$



                Hint: If you don't like the direct route you can go by contradiction: Suppose $a neq a'$, for example. Then use pairing to obtain the sets $ a $ and $ a',b' $. Now use the contrapositive of extensionality; what do you get?







                share|cite|improve this answer












                share|cite|improve this answer



                share|cite|improve this answer










                answered Mar 26 at 18:08









                MacRanceMacRance

                1826




                1826



























                    draft saved

                    draft discarded
















































                    Thanks for contributing an answer to Mathematics Stack Exchange!


                    • Please be sure to answer the question. Provide details and share your research!

                    But avoid


                    • Asking for help, clarification, or responding to other answers.

                    • Making statements based on opinion; back them up with references or personal experience.

                    Use MathJax to format equations. MathJax reference.


                    To learn more, see our tips on writing great answers.




                    draft saved


                    draft discarded














                    StackExchange.ready(
                    function ()
                    StackExchange.openid.initPostLogin('.new-post-login', 'https%3a%2f%2fmath.stackexchange.com%2fquestions%2f3163450%2fhow-to-prove-that-a-a-b-rightarrow-a-a-b%23new-answer', 'question_page');

                    );

                    Post as a guest















                    Required, but never shown





















































                    Required, but never shown














                    Required, but never shown












                    Required, but never shown







                    Required, but never shown

































                    Required, but never shown














                    Required, but never shown












                    Required, but never shown







                    Required, but never shown







                    Popular posts from this blog

                    Lowndes Grove History Architecture References Navigation menu32°48′6″N 79°57′58″W / 32.80167°N 79.96611°W / 32.80167; -79.9661132°48′6″N 79°57′58″W / 32.80167°N 79.96611°W / 32.80167; -79.9661178002500"National Register Information System"Historic houses of South Carolina"Lowndes Grove""+32° 48' 6.00", −79° 57' 58.00""Lowndes Grove, Charleston County (260 St. Margaret St., Charleston)""Lowndes Grove"The Charleston ExpositionIt Happened in South Carolina"Lowndes Grove (House), Saint Margaret Street & Sixth Avenue, Charleston, Charleston County, SC(Photographs)"Plantations of the Carolina Low Countrye

                    random experiment with two different functions on unit interval Announcing the arrival of Valued Associate #679: Cesar Manara Planned maintenance scheduled April 23, 2019 at 00:00UTC (8:00pm US/Eastern)Random variable and probability space notionsRandom Walk with EdgesFinding functions where the increase over a random interval is Poisson distributedNumber of days until dayCan an observed event in fact be of zero probability?Unit random processmodels of coins and uniform distributionHow to get the number of successes given $n$ trials , probability $P$ and a random variable $X$Absorbing Markov chain in a computer. Is “almost every” turned into always convergence in computer executions?Stopped random walk is not uniformly integrable

                    How should I support this large drywall patch? Planned maintenance scheduled April 23, 2019 at 00:00UTC (8:00pm US/Eastern) Announcing the arrival of Valued Associate #679: Cesar Manara Unicorn Meta Zoo #1: Why another podcast?How do I cover large gaps in drywall?How do I keep drywall around a patch from crumbling?Can I glue a second layer of drywall?How to patch long strip on drywall?Large drywall patch: how to avoid bulging seams?Drywall Mesh Patch vs. Bulge? To remove or not to remove?How to fix this drywall job?Prep drywall before backsplashWhat's the best way to fix this horrible drywall patch job?Drywall patching using 3M Patch Plus Primer